Đến nội dung

tuanyeubeo2000 nội dung

Có 64 mục bởi tuanyeubeo2000 (Tìm giới hạn từ 09-05-2020)



Sắp theo                Sắp xếp  

#641473 CMR: $\frac{1}{a+1}+\frac{1}...

Đã gửi bởi tuanyeubeo2000 on 20-06-2016 - 21:30 trong Bất đẳng thức và cực trị

Chứng minh rằng: $\frac{1}{a+1}+\frac{1}{b+1}+\frac{1}{c+1}\leq 1$ với $a,b,c>1$ thỏa mãn: $\frac{1}{a^2-1}+\frac{1}{b^2-1}+\frac{1}{c^2-1}=1$

$ giả\quad thiết\quad <=>\frac { 1 }{ 3 } =\sum { \frac { 1 }{ (3a-3)(a+1) }  } \ge \sum { \frac { 4 }{ { (4a-2) }^{ 2 } } \ge \frac { (\sum { \frac { 1 }{ 2a-1 } ) }  }{ 3 }  } \\ ->1\ge \sum { \frac { 1 }{ 2a-1 }  } .\quad Ta\quad có\quad \frac { 1 }{ 2a-1 } +\frac { 1 }{ 3 } \ge \frac { 4 }{ 2a+2 } =\frac { 2 }{ a+1 } ->2\ge \sum { \frac { 2 }{ a+1 }  } \\ ->\sum { \frac { 1 }{ a+1 } \le 1 }  $




#641431 Tổng hợp các bài BĐT trong các đề thi thử THPT Quốc Gia môn Toán năm 2017

Đã gửi bởi tuanyeubeo2000 on 20-06-2016 - 18:25 trong Bất đẳng thức và cực trị

Lâu rồi mới lên :D

Bài 170 : $ Cho\quad 0\le a,b,c\le 1\quad và\quad a+b+c=2.\quad CMR:\\ P={ a }^{ 3 }+{ b }^{ 3 }+{ c }^{ 3 }+4abc\le \frac { 9 }{ 4 } $

$ từ\quad giả\quad thiết\quad ->\quad 2-2a\ge 0\quad ->b+c-a\ge 0\quad .\quad tương\quad tự\quad ta\quad cũng\quad có\quad a+b-c\ge 0\quad và\quad a+c-b\ge 0\\ Đặt\quad a+b-c=2x\quad ;\quad b+c-a=2y;\quad c+a-b=2z->\quad a=x+y;\quad b=y+z;\quad c=z+x->x+y+z=1\\ bđt\quad đã\quad cho\quad <=>4\sum { { (x+y) }^{ 3 } } +16\prod { (x+y) } \le 9{ (x+y+z) }^{ 3 }\\ <=>\sum { { x }^{ 3 }+22xyz\ge  } \sum { xy(x+y) }( schur) $

P/s : ai cho xin cách ép biên




#641401 Tổng hợp các bài BĐT trong các đề thi thử THPT Quốc Gia môn Toán năm 2017

Đã gửi bởi tuanyeubeo2000 on 20-06-2016 - 15:26 trong Bất đẳng thức và cực trị

Lâu rồi mới lên :D

Bài 170 : $ Cho\quad 0\le a,b,c\le 1\quad và\quad a+b+c=2.\quad CMR:\\ P={ a }^{ 3 }+{ b }^{ 3 }+{ c }^{ 3 }+4abc\le \frac { 9 }{ 4 } $




#640625 Tìm max $P=\frac{1}{\sqrt{x^2+3y^2}...

Đã gửi bởi tuanyeubeo2000 on 16-06-2016 - 07:51 trong Bất đẳng thức và cực trị

Cho $x,y,z>0$. Tìm GTLN của biểu thức

                     $P=\dfrac{1}{\sqrt{x^2+3y^2}}+\dfrac{1}{\sqrt{3x^2+y^2}}-\dfrac{2}{3(x+y)^3}$

trong topic kia giải 1 lần rồi mà anh




#640254 Tổng hợp các bài BĐT trong các đề thi thử THPT Quốc Gia môn Toán năm 2017

Đã gửi bởi tuanyeubeo2000 on 14-06-2016 - 13:06 trong Bất đẳng thức và cực trị

Cho các số dương $x,y,z$ thỏa mãn: $\frac{1}{\sqrt{x}}+\frac{1}{\sqrt{y}}+\frac{1}{\sqrt{z}}=\frac{1}{\sqrt{xyz}}$. Tìm giá trị lớn nhất của biểu thức:

$P=\frac{2\sqrt{x}}{1+x}+\frac{2\sqrt{y}}{1+y}+\frac{z-1}{1+z}$

$ gt\quad <=>\sum { \sqrt { xy } =1.\quad Đặt\quad \sqrt { x } =a;\sqrt { y } =b;\sqrt { z } =c->ab+bc+ca=1.\quad Viết\quad lại\quad : } \\ P=\frac { 2a }{ { a }^{ 2 }+1 } +\frac { 2b }{ { b }^{ 2 }+1 } +\frac { { c }^{ 2 }-1 }{ { c }^{ 2 }+1 } .\quad Ta\quad có\quad :\frac { 2a }{ { a }^{ 2 }+1 } =\frac { 2a }{ (a+b)(a+c) } \\ ->P\le \frac { 4ab+2bc+2ac }{ (a+b)(b+c)(c+a) } +\frac { { c }^{ 2 }-1 }{ { c }^{ 2 }+1 } =\frac { 2(1+ab) }{ (a+b)(b+c)(c+a) } +\frac { { c }^{ 2 }-1 }{ { c }^{ 2 }+1 } \\ \le \frac { 2\sqrt { { (a }^{ 2 }+1)({ b }^{ 2 }+1) }  }{ (a+b)(b+c)(c+a) } +\frac { { c }^{ 2 }-1 }{ { c }^{ 2 }+1 } =\frac { 2(a+b)\sqrt { (a+c)(b+c) }  }{ \prod { (a+b) }  } +\frac { { c }^{ 2 }-1 }{ { c }^{ 2 }+1 } =\frac { 2 }{ \sqrt { (c+a)(c+b) }  } +\frac { { c }^{ 2 }-1 }{ { c }^{ 2 }+1 } \\ =\frac { 2 }{ \sqrt { { c }^{ 2 }+1 }  } +\frac { { c }^{ 2 }-1 }{ { c }^{ 2 }+1 } (\quad hàm\quad c) $




#640075 Chứng minh rằng : $7(ab+bc+ac) \leq 2+9abc$

Đã gửi bởi tuanyeubeo2000 on 13-06-2016 - 16:57 trong Bất đẳng thức và cực trị

Cách đơn giản hơn 1 chút: 

Ta có: $(a+b+c)^2\geq 3(ab+bc+ca)\Rightarrow 3(ab+bc+ca)\leq 1$

Áp dụng BĐT Schur ta có: 

$(a+b+c)^3+9abc\geq 4(a+b+c)(ab+bc+ca)\Rightarrow 4(ab+bc+ca)\leq 1+9abc$

Cộng theo vế 2 BĐT vừa Chứng minh ta được đpcm.

khác cách mình à ?




#640059 $\sum ab^3\geq \sum a^2b^2$

Đã gửi bởi tuanyeubeo2000 on 13-06-2016 - 15:34 trong Bất đẳng thức và cực trị

Tớ nhớ là đọc qua trong stbđt rồi , bị ngược hay sao ý




#640056 $\sum \frac{a}{p-a}\geq \sum...

Đã gửi bởi tuanyeubeo2000 on 13-06-2016 - 15:33 trong Bất đẳng thức và cực trị

Cho a,b,c là chiều dài các cạnh của tam giác có chu vi 2p. Chứng minh rằng:
$\sum \frac{a}{p-a}\geq \sum \sqrt{\frac{b+c}{p-a}}$

$ Không\quad mất\quad tính\quad tổng\quad quát\quad giả\quad sử\quad a\ge b\ge c->p-a\le p-b\le p-c\\ ->\frac { 1 }{ p-a } \ge \frac { 1 }{ p-b } \ge \frac { 1 }{ p-c } .\quad Nên\quad áp\quad dụng\quad chebyshev\quad ta\quad được\quad :\\ \sum { \frac { a }{ p-a } \ge \frac { (a+b+c)(\frac { 1 }{ p-a } +\frac { 1 }{ p-b } +\frac { 1 }{ p-c } ) }{ 3 } (*)\\  } \quad \\ Lại\quad có\quad :\sum { \sqrt { \frac { b+c }{ p-a }  } \le \sqrt { (2a+2b+2c)(\frac { 1 }{ p-a } +\frac { 1 }{ p-b } +\frac { 1 }{ p-c } ) } (**)\quad  } \\ BĐT\quad đã\quad cho\quad <=>\frac { (a+b+c)(\frac { 1 }{ p-a } +\frac { 1 }{ p-b } +\frac { 1 }{ p-c } ) }{ 3 } \ge \sqrt { (2a+2b+2c)(\frac { 1 }{ p-a } +\frac { 1 }{ p-b } +\frac { 1 }{ p-c } ) } \\ Hay\quad :(a+b+c)(\frac { 1 }{ p-a } +\frac { 1 }{ p-b } +\frac { 1 }{ p-c } )\ge 18.\quad Dễ\quad thấy\quad \\ (a+b+c)(\frac { 1 }{ p-a } +\frac { 1 }{ p-b } +\frac { 1 }{ p-c } )=(a+b+c)(\frac { 2 }{ a+b-c } +\frac { 2 }{ b+c-a } +\frac { 2 }{ c+a-b } )\ge \frac { 18(a+b+c) }{ a+b+c } =18\\ ->dfcm $




#640048 Chứng minh rằng : $7(ab+bc+ac) \leq 2+9abc$

Đã gửi bởi tuanyeubeo2000 on 13-06-2016 - 15:16 trong Bất đẳng thức và cực trị

Help me!!!!!!!!!!!!!

$ Đặt\quad a+b+c=q=1\quad ,\quad ab+bc+ca=q\quad ,\quad abc=r\quad .\quad Theo\quad schur\quad bậc\quad 3\quad ta\quad có\quad \\ 9r\ge 4pq-{ p }^{ 3 }=4q-1.\quad Ta\quad sẽ\quad chứng\quad minh\quad 4q-1+2\ge 7q<=>1\ge 3q\quad đúng\quad vì\quad \\ { (a+b+c) }^{ 2 }\ge 3(ab+bc+ca)->q\le \frac { 1 }{ 3 } ->dfcm $




#640045 $\sum ab^3\geq \sum a^2b^2$

Đã gửi bởi tuanyeubeo2000 on 13-06-2016 - 15:12 trong Bất đẳng thức và cực trị

ngược dấu rồi




#639963 Tổng hợp các bài BĐT trong các đề thi thử THPT Quốc Gia môn Toán năm 2017

Đã gửi bởi tuanyeubeo2000 on 13-06-2016 - 02:15 trong Bất đẳng thức và cực trị

Bài 156

Cho x, y, z là các số thực không âm thoả mãn:  $\sqrt{1+2x}+\sqrt{1+2y}+\sqrt{1+z^2}=5$

Tìm giá trị lớn nhất của $P=x^3+y^3+2z^3$

$ Ta\quad có\quad :\quad \sqrt { 1+2x } +\sqrt { 1+2y } \ge 1+\sqrt { 1+2(x+y) } \\ ->4\ge \sqrt { 1+{ z }^{ 2 } } +\sqrt { 1+2(x+y) } \ge 1+\sqrt { 1+{ z }^{ 2 }+2(x+y) } ->{ z }^{ 2 }+2(x+y)\le 8->\begin{cases} x+y\le 4 \\ { z }^{ 2 }\le 8<16 \end{cases}\\ ->\begin{cases} x+y\le 4 \\ 2{ z }^{ 2 }(4-z)\ge 0 \end{cases}<=>\begin{cases} x+y\le 4 \\ { 2 }z^{ 3 }\le { 8z }^{ 2 } \end{cases}\\$.

$ Lại\quad có\quad :  ={ x }^{ 3 }+{ y }^{ 3 }+{ 2z }^{ 3 }\le { (x+y) }^{ 3 }+{ 2z }^{ 3 }\le 8(2x+2y)+8{ z }^{ 2 }=8({ z }^{ 2 }+2(x+y))=64 $




#639960 CMR: $\frac{a+b+c}{3}\geq \sqrt[5]...

Đã gửi bởi tuanyeubeo2000 on 13-06-2016 - 01:19 trong Bất đẳng thức và cực trị

Cho a,b,c là độ dài 3 cạnh của tam giác thỏa mãn: $(a+b-c)(b+c-a)(c+a-b)=1$

Chứng minh rằng: $\frac{a+b+c}{3}\geq \sqrt[5]{\frac{a^2+b^2+c^2}{3}}$

$ Ta\quad có\quad (a+b-c)(b+c-a)(c+a-b)\le 1->{ (ab+bc+ca) }^{ 2 }\ge 3abc(a+b+c)\ge 3(a+b+c)\\ hay\quad \frac { a+b+c }{ { (ab+bc+ca) }^{ 2 } } \le \frac { 1 }{ 3 } .Lại\quad có\quad { (ab+bc+ca) }^{ 2 }({ a }^{ 2 }+{ b }^{ 2 }+{ c }^{ 2 })\le \frac { { (a+b+c) }^{ 6 } }{ 27 } \\ ->({ a }^{ 2 }+{ b }^{ 2 }+{ c }^{ 2 })\le \frac { { (a+b+c) }^{ 5 } }{ 81 } ->\frac { ({ a }^{ 2 }+{ b }^{ 2 }+{ c }^{ 2 }) }{ 3 } \le { (\frac { a+b+c }{ 3 } ) }^{ 5 }->dfcm $




#639938 Tổng hợp các bài BĐT trong các đề thi thử THPT Quốc Gia môn Toán năm 2017

Đã gửi bởi tuanyeubeo2000 on 12-06-2016 - 23:00 trong Bất đẳng thức và cực trị

Bài 155:  $ Cho\quad x,y,z>0\quad thõa\quad mãn\quad :\quad { x }^{ 2 }+{ 8y }^{ 2 }+{ 8z }^{ 2 }=20.\quad Tìm\quad GTLN:\\ P=\frac { { 2x }^{ 2 } }{ { x }^{ 2 }(4-yz)+8 } +\frac { y+z }{ x+y+z+1 } -\frac { { x }^{ 2 }+{ (y+z) }^{ 2 } }{ 100 }  $




#639934 Tổng hợp các bài BĐT trong các đề thi thử THPT Quốc Gia môn Toán năm 2017

Đã gửi bởi tuanyeubeo2000 on 12-06-2016 - 22:53 trong Bất đẳng thức và cực trị

ta chú ý đến bổ đề sau: với x,y,z là các số thực dương  thỏa mãn x+y+z=3 thì ta có $xy+yz+zx\leq \sqrt{x}+\sqrt{y}+\sqrt{z}$

Chứng minh: ta có

$x^{2}+\sqrt{x}+\sqrt{x}\geq 3x$ 

tương tự các biến còn lại ta có

$x^{2}+y^{2}+z^{2}+2\sqrt{x}+2\sqrt{y}+2\sqrt{z}\geq 3(x+y+z)=(x+y+z)^{2}= x^{2}+y^{2}+z^{2}+2(xy+yz+zx)\Leftrightarrow \sqrt{x}+\sqrt{y}+\sqrt{z}\geq xy+yz+zx$

Quay trở lại bài toán ta sẽ có

$P\geq \frac{16}{\sqrt{x+y+z+1}}+\frac{\frac{(x+y+z)^{2}-3}{2}+1}{x+y+z}= \frac{16}{\sqrt{x+y+z+1}}+\frac{(x+y+z)^{2}-1}{2(x+y+z)}$

Đến đây đặt t=x+y+z với t$\in (0;3]$ sau đó xét hàm là xong

bổ đề hay vậy mà tớ quên hihi




#639863 Tổng hợp các bài BĐT trong các đề thi thử THPT Quốc Gia môn Toán năm 2017

Đã gửi bởi tuanyeubeo2000 on 12-06-2016 - 17:15 trong Bất đẳng thức và cực trị

 

 

Bài 149: Cho các số thực dương $x,y,z$. Tìm giá trị nhỏ nhất của biểu thức:

 

$$P=\frac{x}{x+2z}+\frac{y}{y+2z}+\frac{2\left ( z^{2}-xy \right )}{\left ( x+y+z \right )^{2}}$$

$ \frac { x }{ x+2z } +\frac { y }{ y+2z } \ge \frac { { (x+y) }^{ 2 } }{ { (x+y) }^{ 2 }+2z(x+y)+{ z }^{ 2 }-({ z }^{ 2 }+2xy) } =\frac { { (x+y) }^{ 2 } }{ { (x+y+z) }^{ 2 }+{ 2(z }^{ 2 }-xy)-3{ z }^{ 2 } } \\ =\frac { 1 }{ { (1+\frac { z }{ x+y } ) }^{ 2 }+\frac { 2({ z }^{ 2 }-xy) }{ { (x+y) }^{ 2 } } -3{ \left( \frac { z }{ x+y }  \right)  }^{ 2 } } .Nên\\ P\ge \frac { 1 }{ { (1+\frac { z }{ x+y } ) }^{ 2 }+\frac { 2({ z }^{ 2 }-xy) }{ { (x+y) }^{ 2 } } -3{ \left( \frac { z }{ x+y }  \right)  }^{ 2 } } +\frac { \frac { 2{ (z }^{ 2 }-xy) }{ { (x+y) }^{ 2 } }  }{ { (1+\frac { z }{ x+y } ) }^{ 2 } } .\quad Đặt\quad A=\frac { z }{ x+y } ;\quad B=\frac { 2{ (z }^{ 2 }-xy) }{ { (x+y) }^{ 2 } } \\ Ta\quad có\quad :\quad B\ge 2{ \left( \frac { z }{ x+y }  \right)  }^{ 2 }-\frac { 1 }{ 2 } \ge \frac { 2z }{ x+y } -1=2A-1->B+1\ge 2A.\quad Viết\quad lại\quad :\\ P\le \frac { 1 }{ { (A+1) }^{ 2 }+B-3{ A }^{ 2 } } +\frac { B }{ { (A+1) }^{ 2 } } $

-> Hướng này chắc sai , các bác xem qua cho em cái nhé , em làm vội




#639859 Tổng hợp các bài BĐT trong các đề thi thử THPT Quốc Gia môn Toán năm 2017

Đã gửi bởi tuanyeubeo2000 on 12-06-2016 - 16:48 trong Bất đẳng thức và cực trị

Bài 154: $ Cho:\quad x,y,z>0\quad thõa\quad mãn\quad :\quad \sum { { x }^{ 2 } } =3\quad .Tìm\quad GTNN:\\ P=\frac { 16 }{ \sqrt { (\sum { { x }^{ 2 }{ y }^{ 2 } } )+1 }  } +\frac { xy+yz+zx+1 }{ x+y+z } $




#639857 Tổng hợp các bài BĐT trong các đề thi thử THPT Quốc Gia môn Toán năm 2017

Đã gửi bởi tuanyeubeo2000 on 12-06-2016 - 16:42 trong Bất đẳng thức và cực trị

 

Bài 152:Cho các số thực dương x,y,z thỏa mãn $x^{2}+z^{2}\leq 2$.Tìm GTNN của biểu thức

P=$\frac{x^{2}+y^{2}}{z^{2}+xy}+\frac{y^{2}+z^{2}}{x^{2}+yz}-\frac{40}{3}\sqrt{\frac{(x+y+z)^{2}}{x^{2}+y^{2}+z^{2}+1}+4}$

 

$ Ta\quad có:\quad 2({ x }^{ 2 }+{ y }^{ 2 }+{ z }^{ 2 }+1)\ge 3({ x }^{ 2 }+{ z }^{ 2 })+2{ y }^{ 2 }\ge { 2(x }^{ 2 }+{ z }^{ 2 }+xy+yz)\\ Lại\quad có:\frac { { x }^{ 2 }+{ y }^{ 2 } }{ { z }^{ 2 }+xy } +\frac { { y }^{ 2 }+{ z }^{ 2 } }{ { x }^{ 2 }+yz } \ge \frac { { (x+z) }^{ 2 } }{ { x }^{ 2 }+{ z }^{ 2 }+xy+yz } +\frac { { 2y }^{ 2 } }{ { x }^{ 2 }+{ z }^{ 2 }+xy+yz } \\ =\frac { { (x+z) }^{ 2 }+{ 2y }^{ 2 } }{ { x }^{ 2 }+{ z }^{ 2 }+xy+yz } \ge \frac { 2{ (x+y+z) }^{ 2 } }{ 3({ x }^{ 2 }+{ z }^{ 2 }+xy+yz) } .Nên\quad :\\ P\ge \frac { 2{ (x+y+z) }^{ 2 } }{ 3({ x }^{ 2 }+{ z }^{ 2 }+xy+yz) } -\frac { 40 }{ 3 } \sqrt { \frac { { (x+y+z) }^{ 2 } }{ ({ x }^{ 2 }+{ z }^{ 2 }+xy+yz) } +4 } (Đặt\quad t=\frac { { (x+y+z) }^{ 2 } }{ ({ x }^{ 2 }+{ z }^{ 2 }+xy+yz) } )\\ Khảo\quad sát\quad hàm\quad nữa $




#638921 Tổng hợp các bài BĐT trong các đề thi thử THPT Quốc Gia môn Toán năm 2017

Đã gửi bởi tuanyeubeo2000 on 08-06-2016 - 14:26 trong Bất đẳng thức và cực trị

Bài144: Cho các số thực dương x,y,z thỏa mãn điều kiện $x^{2}+y^{2}+z^{2}=1$.Tim min:

$P=4xy+4yz+10xz+\frac{1}{(x+z)(x+y+z)+1}$

có min thật à 




#638919 Tổng hợp các bài BĐT trong các đề thi thử THPT Quốc Gia môn Toán năm 2017

Đã gửi bởi tuanyeubeo2000 on 08-06-2016 - 14:14 trong Bất đẳng thức và cực trị

Gần tương tự nè . Bài 146 : $ Cho\quad a,b,c\ge 0\quad thõa\quad mãn\quad a+b+c=3\quad .\quad CMR:\\ \sum { \frac { 2 }{ { a }^{ 3 } }  } +\sum { \frac { 1 }{ { a }^{ 2 }-ab+{ b }^{ 2 } }  } \ge 9 $

 




#638825 Tổng hợp các bài BĐT trong các đề thi thử THPT Quốc Gia môn Toán năm 2017

Đã gửi bởi tuanyeubeo2000 on 07-06-2016 - 23:23 trong Bất đẳng thức và cực trị

Thấy pic yên ắng quá .

Bài 140 : $ Cho\quad a,b,c\ge 0\quad thõa\quad mãn\quad a+b+c=3\quad .\quad CMR:\\ \sum { \frac { 2 }{ { a }^{ 2 } }  } +\sum { \frac { 1 }{ { a }^{ 2 }-ab+{ b }^{ 2 } }  } \ge 9 $




#638734 Tìm GTLN của $S=a^2+b^2+c^2$

Đã gửi bởi tuanyeubeo2000 on 07-06-2016 - 16:35 trong Bất đẳng thức và cực trị

Mọi người bài này trâu lắm nè, làm đủ cách cũng ko ra thôi đành nhờ m.n giúp. Help me!!!  :ukliam2:  :ukliam2:  :ukliam2:

$ Đặt\quad { a }^{ 2 }=x;{ b }^{ 2 }=y;c^{ 2 }=z(x.y,z\ge 0).->\sum { { x }^{ 3 } } =3.\quad Và\quad P=\sum { x } .\\ Áp\quad dụng\quad holder\quad ta\quad \quad có\quad :\quad \quad \quad (\sum { { x }^{ 3 } } ).3.3\ge { (x+y+z) }^{ 3 }->x+y+z\le 3->P\le 3$




#638684 đề thi tuyển sinh vào 10 chuyên toán Lam Sơn Thanh Hóa 2016 (vòng 2)

Đã gửi bởi tuanyeubeo2000 on 07-06-2016 - 11:53 trong Tài liệu - Đề thi

ù bạn

thật à :v




#638612 đề thi tuyển sinh vào 10 chuyên toán Lam Sơn Thanh Hóa 2016 (vòng 2)

Đã gửi bởi tuanyeubeo2000 on 06-06-2016 - 22:21 trong Tài liệu - Đề thi

chừa anh đi

sai à :v




#638595 đề thi tuyển sinh vào 10 chuyên toán Lam Sơn Thanh Hóa 2016 (vòng 2)

Đã gửi bởi tuanyeubeo2000 on 06-06-2016 - 20:54 trong Tài liệu - Đề thi

 

KỲ THI TUYỂN SINH VÀO 10 CHUYÊN LAM SƠN

Năm học 2016 - 2017

 

Môn thi: TOÁN

( Dành cho thí sinh thi vào lớp 10 chuyên Toán)

Thời gian: 150 phút (không kể thời gian giao đề)

Ngày thi: 06/06/2016

(Đề thi có 01 trang, gồm 05 câu)

 

Câu 1: (2 điểm)

a/ chứng minh rằng: $\frac{1}{2\sqrt{1}}+\frac{1}{3\sqrt{2}}+...+\frac{1}{2016\sqrt{2015}}> \frac{1931}{1975}$

Câu 3: ( 2điểm)

a/ tìm nghiệm nguyên (x;y) của phương trình: $(x+1)(x+2)(x+8)(x+9)=y^{2}$

 

 

$ Câu 1 : Ta\quad có\quad \frac { 1 }{ 2\sqrt { 1 }  } +\frac { 1 }{ 3\sqrt { 2 }  } +...+\frac { 1 }{ 2016\sqrt { 2015 }  } >\frac { 1 }{ 1.2 } +\frac { 1 }{ 2.3 } +...+\frac { 1 }{ 2015.2016 } =1-\frac { 1 }{ 2016 } =\frac { 2015 }{ 2016 } >\frac { 1931 }{ 1975 } $ 

$ Câu 3a :<=>({ x }^{ 2 }+10x+9)({ x }^{ 2 }+10+16)={ y }^{ 2 }.Đặt\quad :{ x }^{ 2 }+10x+9=a->a(a+7)={ y }^{ 2 }(x,y,a\in Z)\\ <=>(y-a)(y+a)=7a.\\ +)TH1:y-a\vdots a->đặt\quad y-a=ka(k\in Z)->a({ ak }^{ 2 }-a-7)=0\\ .Nếu:a=0->x=y=0\\ .Nếu:{ ak }^{ 2 }-a-7=0<=>a({ k }^{ 2 }-1)=7\quad ->\quad xét\quad ước\\ +)\quad TH2:Xét\quad tương\quad tự\quad cho\quad y+a $ 

p/s : 3a hình như giải sai :v




#638551 P=$\frac{x}{x+1}+\frac{y}{y...

Đã gửi bởi tuanyeubeo2000 on 06-06-2016 - 18:37 trong Bất đẳng thức và cực trị

Có phải bạn dùng cô-si biến thê 1/x+1/y>=4/(x+y) ko 

chuẩn rồi